Uniqueness of Solution for $\Delta u=0$ in a Ball with Boundary Condition $\phi$

  • MHB
  • Thread starter evinda
  • Start date
  • Tags
    Uniqueness
In summary, the conversation discusses the uniqueness of a solution to a problem involving the Laplace equation and a given boundary condition. It is proven through the use of a theorem and the definition of a function P. The conversation also delves into the properties of the integral and the continuity of the given function. Further explanations are provided for the properties and the reasoning behind them.
  • #1
evinda
Gold Member
MHB
3,836
0
Hello! (Wave)


Let $(\star)\left\{\begin{matrix}
\Delta u=0 & \text{ in } B_R \\
u|_{\partial{B_R}}=\phi &
\end{matrix}\right.$.

Theorem: If $\phi \in C^0(\partial{B_R})$ then there is a unique solution of the problem $(\star)$ and $u(x)=\frac{R^2-|x|^2}{w_n R} \int_{\partial{B_R}} \frac{\phi(\xi) dS}{|x- \xi|^n}$.

Proof: if $x_0 \in \partial{B_R}$ then it has to hold that $\lim_{x \to x_0 } u(x)=\phi(x_0)$.

$P(x, \xi)=\frac{R^2-|x|^2}{w_n R |x-\xi|^n}$

It holds that $\int_{\partial{B_R}} P(x, \xi) dS=1$.

$$\left| u(x)- \phi(x_0)\right|=\left| \int_{\partial{B_R}} P(x, \xi) (\phi(\xi)-\phi(x_0)) dS\right| \leq \int_{\partial{B_R}} P(x, \xi) |\phi(\xi)-\phi(x_0)| dS= \int_{|\xi-x_0| \leq \delta} P(x, \xi)|\phi(\xi)-\phi(x_0)| dS+\int_{|\xi-x_0| > \delta} P(x, \xi)|\phi(\xi)-\phi(x_0)| dS $$

Let $I_1=\int_{|\xi-x_0| \leq \delta} P(x, \xi)|\phi(\xi)-\phi(x_0)| dS$ and $I_2=\int_{|\xi-x_0| > \delta} P(x, \xi)|\phi(\xi)-\phi(x_0)| dS$.

$\forall \epsilon >0 \exists \delta>0$ so that if $|\xi-x_0| \leq \delta$ then $|I_1|< \frac{\epsilon}{2}$.

$I_2 \leq 2M \frac{R^2-|x|^2}{w_n R \delta^n} \int_{|\xi-x_0|=R} dS=2M \frac{R^2-|x|^2}{\delta^n} R^{n-2} \leq \frac{\epsilon}{2}$ if $x$ is near to $ x_0 $.

So we have the following: $\forall \epsilon>0 \exists \delta>0$ such that $|u(x)-\phi(x_0)| \leq \epsilon \Leftrightarrow \lim_{x \to x_0} u(x)=\phi(x_0)$.

First of all, why does it hold that $\left| u(x)- \phi(x_0)\right|=\left| \int_{\partial{B_R}} P(x, \xi) (\phi(\xi)-\phi(x_0)) dS\right| $ and not $\left| u(x)- \phi(x_0)\right|=\left| \int_{\partial{B_R}} ( P(x, \xi) \phi(\xi)-\phi(x_0) ) dS\right|$ ?Also why does it hold that $\int_{\partial{B_R}} P(x, \xi) |\phi(\xi)-\phi(x_0)| dS= \int_{|\xi-x_0| \leq \delta} P(x, \xi)|\phi(\xi)-\phi(x_0)| dS+\int_{|\xi-x_0| > \delta} P(x, \xi)|\phi(\xi)-\phi(x_0)| dS$ ?

Could you also explain to me how we get the inequalities for $I_1$ and $I_2$ ?
 
Physics news on Phys.org
  • #2
Hey evinda! (Smile)

evinda said:
First of all, why does it hold that $\left| u(x)- \phi(x_0)\right|=\left| \int_{\partial{B_R}} P(x, \xi) (\phi(\xi)-\phi(x_0)) dS\right| $ and not $\left| u(x)- \phi(x_0)\right|=\left| \int_{\partial{B_R}} ( P(x, \xi) \phi(\xi)-\phi(x_0) ) dS\right|$ ?

Isn't:
$$\int_{\partial{B_R}} \phi(x_0)dS = w_nR^{n-1}\phi(x_0) \ne \phi(x_0)$$
(Wondering)

While:
$$\phi(x_0) = u(x_0) = \int_{\partial{B_R}} P(x_0,\xi) \phi(x_0)dS
$$

Also why does it hold that $\int_{\partial{B_R}} P(x, \xi) |\phi(\xi)-\phi(x_0)| dS= \int_{|\xi-x_0| \leq \delta} P(x, \xi)|\phi(\xi)-\phi(x_0)| dS+\int_{|\xi-x_0| > \delta} P(x, \xi)|\phi(\xi)-\phi(x_0)| dS$ ?

We can split up the domain of an integral into 2 subsets that combine together to the domain.
Compare for instance:
$$\int_0^1 f(x)dx = \int_0^\delta f(x)dx + \int_\delta^1 f(x)dx$$
Note that we're still constrained to $\partial{B_R}$. (Nerd)

Could you also explain to me how we get the inequalities for $I_1$ and $I_2$ ?

Let's start with $I_1$.

$$|I_1|=\left|\int_{|\xi-x_0| \leq \delta} P(x, \xi)|\phi(\xi)-\phi(x_0)| dS\right| \\
\le \sup_{|\xi-x_0| \leq \delta} |P(x,\xi)| \cdot \sup_{|\xi-x_0| \leq \delta} |\phi(\xi)-\phi(x_0)| \cdot \int_{\partial{B_R}}dS \\
\le M \cdot \epsilon' \cdot w_nR^{n-1} = \frac\epsilon 2
$$
where $\epsilon'$ is introduced since $\phi$ is given to be continuous.
So given an $\epsilon$ we can find an $\epsilon'$ and from the continuity of $\phi$, we can find the appropriate $\delta$.

Note that we can only guarantee the bound on $|P(x, \xi)|$ if $x\notin \partial{B_R}$, which is given since $x\in B_R$. (Thinking)
 
  • #3
I like Serena said:
Isn't:
$$\int_{\partial{B_R}} \phi(x_0)dS = w_nR^{n-1}\phi(x_0) \ne \phi(x_0)$$
(Wondering)

While:
$$\phi(x_0) = u(x_0) = \int_{\partial{B_R}} P(x_0,\xi) \phi(x_0)dS
$$

Oh yes, right... (Nod)

I like Serena said:
We can split up the domain of an integral into 2 subsets that combine together to the domain.
Compare for instance:
$$\int_0^1 f(x)dx = \int_0^\delta f(x)dx + \int_\delta^1 f(x)dx$$
Note that we're still constrained to $\partial{B_R}$. (Nerd)

Ok...

I like Serena said:
Let's start with $I_1$.

$$|I_1|=\left|\int_{|\xi-x_0| \leq \delta} P(x, \xi)|\phi(\xi)-\phi(x_0)| dS\right|
\le \sup_{|\xi-x_0| \leq \delta} |P(x,\xi)| \cdot \sup_{|\xi-x_0| \leq \delta} |\phi(\xi)-\phi(x_0)| \cdot \int_{\partial{B_R}}dS
\le M \cdot \epsilon' \cdot w_nR^{n-1} = \frac\epsilon 2
$$
where $\epsilon'$ is introduced since $\phi$ is given to be continuous.

I see.

I like Serena said:
So given an $\epsilon$ we can find an $\epsilon'$ and from the continuity of $\phi$, we can find the appropriate $\delta$.

Don't we have from the continuity of $\phi$ that $\forall \epsilon'>0 \exists \delta>0$ such that if $|\xi-x_0| \leq \delta$ then $|\phi(\xi)-\phi(x_0)| \leq \epsilon'$ and then we set $\epsilon=2M \epsilon' w_n R^{n-1}$ ? Or am I wrong? (Thinking)

How can find the appropriate $\delta$?

I like Serena said:
Note that we can only guarantee the bound on $|P(x, \xi)|$ if $x\notin \partial{B_R}$, which is given since $x\in B_R$. (Thinking)

Could you explain to me further why $|P(x, \xi)|$ is only bounded if $x\notin \partial{B_R}$?
 
  • #4
evinda said:
Don't we have from the continuity of $\phi$ that $\forall \epsilon'>0 \exists \delta>0$ such that if $|\xi-x_0| \leq \delta$ then $|\phi(\xi)-\phi(x_0)| \leq \epsilon'$ and then we set $\epsilon=2M \epsilon' w_n R^{n-1}$ ? Or am I wrong? (Thinking)

How can find the appropriate $\delta$?

We just did. (Thinking)

For any $\epsilon>0$ we define $\epsilon'=\frac{\epsilon}{2M w_n R^{n-1}}$.
From the continuity of $\phi$ we know that for this $\epsilon'>0$ we can find a $\delta>0$, such that if $|\xi-x_0| <\delta$ then $|\phi(\xi)-\phi(x_0)| < \epsilon'$.
It follows that for this same $\delta$ we now also have $|I_1|<\frac\epsilon 2$.
Could you explain to me further why $|P(x, \xi)|$ is only bounded if $x\notin \partial{B_R}$?

Because $P(x,\xi)=\frac{R^2-|x|^2}{w_nR |x-\xi|^n}$, which is not defined if $x=\xi$.
But that can't be because $x$ is an element of the interior of the ball, and $\xi$ is on the boundary of the ball. (Nerd)
 
  • #5
I like Serena said:
We just did. (Thinking)

For any $\epsilon>0$ we define $\epsilon'=\frac{\epsilon}{2M w_n R^{n-1}}$.
From the continuity of $\phi$ we know that for this $\epsilon'>0$ we can find a $\delta>0$, such that if $|\xi-x_0| <\delta$ then $|\phi(\xi)-\phi(x_0)| \leq \epsilon'$.
It follows that for this same $\delta$ we now also have $|I_1|<\frac\epsilon 2$.

I understand.

I like Serena said:
Because $P(x,\xi)=\frac{R^2-|x|^2}{w_nR |x-\xi|^n}$, which is not defined if $x=\xi$.
But that can't be because $x$ is an element of the interior of the ball, and $\xi$ is on the boundary of the ball. (Nerd)
Do we assume that $x$ is an element of the interior of the ball, and $\xi$ is on the boundary of the ball? Or how do we know it? (Worried)

I thought that we would have at the boundary $|\xi-x|=R$. So do we have $|\xi-x|=0$ ?
 
  • #6
evinda said:
Do we assume that $x$ is an element of the interior of the ball, and $\xi$ is on the boundary of the ball? Or how do we know it? (Worried)

The given formula for $u(x)$ has an integral in it that is not defined if $x$ is on the boundary.
That's because it has $|x-\xi|$ in the denominator, which would become zero when $\xi=x$ ($\xi$ is always a point on the boundary).
Therefore we have to conclude that $x$ is intended to be a point in the interior of the ball. (Thinking)
I thought that we would have at the boundary $|\xi-x|=R$. So do we have $|\xi-x|=0$ ?

$x$ is an arbitrary point inside $B_R$, which is the ball with center $0$ and radius $R$, while $\xi$ is a point on the boundary.
So we have $|\xi - 0|=R$ and $0<|\xi-x|<2R$. (Thinking)
 
  • #7
I like Serena said:
The given formula for $u(x)$ has an integral in it that is not defined if $x$ is on the boundary.
That's because it has $|x-\xi|$ in the denominator, which would become zero when $\xi=x$ ($\xi$ is always a point on the boundary).
Therefore we have to conclude that $x$ is intended to be a point in the interior of the ball. (Thinking)

I see.

I like Serena said:
$x$ is an arbitrary point inside $B_R$, which is the ball with center $0$ and radius $R$, while $\xi$ is a point on the boundary.So we have $|\xi - 0|=R$ and $0<|\xi-x|<2R$. (Thinking)
Ok. So we have

$$\sup_{|\xi-x_0| \leq \delta} |P(x,\xi)|=\sup_{|\xi-x_0| \leq \delta} \left| \frac{R^2-|x|^2}{w_n R |x-\xi|^n} \right|=\frac{R}{w_n} \sup_{|\xi-x_0| \leq \delta} \frac{1}{|x-\xi|^n}$$

Since $|x-\xi|>0$ we have that $\frac{1}{|x-\xi|^n} \to 0$ while $n \to +\infty$ and so $\sup_{|\xi-x_0| \leq \delta} |P(x,\xi)| \to 0$ and so it is bounded. Right? (Thinking)
 
  • #8
evinda said:
Ok. So we have

$$\sup_{|\xi-x_0| \leq \delta} |P(x,\xi)|=\sup_{|\xi-x_0| \leq \delta} \left| \frac{R^2-|x|^2}{w_n R |x-\xi|^n} \right|=\frac{R}{w_n} \sup_{|\xi-x_0| \leq \delta} \frac{1}{|x-\xi|^n}$$

Since $|x-\xi|>0$ we have that $\frac{1}{|x-\xi|^n} \to 0$ while $n \to +\infty$ and so $\sup_{|\xi-x_0| \leq \delta} |P(x,\xi)| \to 0$ and so it is bounded. Right? (Thinking)

Erm... shouldn't we pick $n$ to be some unknown constant? I thought it wasn't supposed to go to infinity. Or is it? (Wondering)

We do have that there is some $\epsilon > 0$ such that $|x-\xi|>\epsilon >0$.
That's because $x$ is supposed to be inside the ball, while $\xi$ is supposed to be on the boundary of the ball.
Isn't therefore:
$$\sup_{|\xi-x_0| \leq \delta} \frac{1}{|x-\xi|^n} < \frac{1}{\epsilon^n}$$
(Wondering)
 
  • #9
I like Serena said:
Erm... shouldn't we pick $n$ to be some unknown constant? I thought it wasn't supposed to go to infinity. Or is it? (Wondering)

Oh yes, you are right.

I like Serena said:
We do have that there is some $\epsilon > 0$ such that $|x-\xi|>\epsilon >0$.
That's because $x$ is supposed to be inside the ball, while $\xi$ is supposed to be on the boundary of the ball.
Isn't therefore:
$$\sup_{|\xi-x_0| \leq \delta} \frac{1}{|x-\xi|^n} < \frac{1}{\epsilon^n}$$
(Wondering)

I see... But $\frac{1}{\epsilon^n}$ isn't bounded for any $\epsilon>0$. So how can we show that $\sup_{|\xi-x_0| \leq \delta} |P(x,\xi)|$ is bounded? (Thinking)
 
  • #10
evinda said:
I see... But $\frac{1}{\epsilon^n}$ isn't bounded for any $\epsilon>0$. So how can we show that $\sup_{|\xi-x_0| \leq \delta} |P(x,\xi)|$ is bounded? (Thinking)

Not for any $\epsilon>0$... (Thinking)
For any given $x$ in the inside of the ball there exists an $\epsilon>0$ such that for each $\xi$ on the boundary with $|\xi - x_0|\le\delta$ we have $|\xi - x|>\epsilon$.

So for every given $x$ in the inside of the ball $\sup_{|\xi-x_0| \leq \delta} |P(x,\xi)|$ is bounded. (Thinking)
 
  • #11
I like Serena said:
Not for any $\epsilon>0$... (Thinking)
For any given $x$ in the inside of the ball there exists an $\epsilon>0$ such that for each $\xi$ on the boundary with $|\xi - x_0|\le\delta$ we have $|\xi - x|>\epsilon$.

So for every given $x$ in the inside of the ball $\sup_{|\xi-x_0| \leq \delta} |P(x,\xi)|$ is bounded. (Thinking)

Ah I see... (Nod)

Can you also explain to me how we get the inequality for $I_2$ ? (Thinking)
 
  • #12
evinda said:
$P(x, \xi)=\frac{R^2-|x|^2}{w_n R |x-\xi|^n}$

$I_2=\int_{|\xi-x_0| > \delta} P(x, \xi)|\phi(\xi)-\phi(x_0)| dS$.

$\forall \epsilon >0 \exists \delta>0$ so that if $|\xi-x_0| \leq \delta$ then $|I_1|< \frac{\epsilon}{2}$.

$I_2 \leq 2M \frac{R^2-|x|^2}{w_n R \delta^n} \int_{|\xi-x_0|=R} dS=2M \frac{R^2-|x|^2}{\delta^n} R^{n-2} \leq \frac{\epsilon}{2}$ if $x$ is near to $ x_0 $.

evinda said:
Ah I see... (Nod)

Can you also explain to me how we get the inequality for $I_2$ ? (Thinking)

We can write:
$$|I_2|=\left|\int_{\xi \in \partial B_R,\ |\xi-x_0| > \delta} P(x, \xi)|\phi(\xi)-\phi(x_0)| dS\right| \\
\le \sup_{\xi \in \partial B_R,\ |\xi-x_0| > \delta} |P(x, \xi)| \cdot \sup_{\xi \in \partial B_R,\ |\xi-x_0| > \delta} |\phi(\xi)-\phi(x_0)| \cdot \left|\int_{\xi \in \partial B_R,\ |\xi-x_0| > \delta}dS\right|
$$
How might we continue? (Wondering)
 
  • #13
How can we get a relation for the integral $\int_{\xi \in \partial B_R,\ |\xi-x_0| > \delta}dS$ ?

Also we have that $\int_{\partial{B_R}} P(x,\xi) ds_{\xi}=1$. Does this imply that $\int_{|\xi-x_0|> \delta, \xi \in \partial{B_R}} P(x,\xi)ds=1$ ? (Thinking)
 
  • #14
evinda said:
How can we get a relation for the integral $\int_{\xi \in \partial B_R,\ |\xi-x_0| > \delta}dS$ ?

Also we have that $\int_{\partial{B_R}} P(x,\xi) ds_{\xi}=1$. Does this imply that $\int_{|\xi-x_0|> \delta, \xi \in \partial{B_R}} P(x,\xi)ds=1$ ? (Thinking)

Don't we have that:
$$\int_{\xi \in \partial B_R,\ |\xi-x_0| > \delta}dS \le \int_{\xi \in \partial B_R}dS = w_n R^{n-1}$$
(Wondering)Btw, I don't understand yet why it's supposed to hold that $\int_{\partial{B_R}} P(x,\xi) ds_{\xi}=1$.
Do you?
Is it somewhere in your notes?
Anyway, if it holds, then, since $P(x,\xi) > 0$, we have:
$$\int_{|\xi-x_0|> \delta, \xi \in \partial{B_R}} P(x,\xi)ds \le \int_{\partial{B_R}} P(x,\xi) ds_{\xi} = 1$$
It seems we don't need it though. (Thinking)
 
  • #15
I like Serena said:
Don't we have that:
$$\int_{\xi \in \partial B_R,\ |\xi-x_0| > \delta}dS \le \int_{\xi \in \partial B_R}dS = w_n R^{n-1}$$
(Wondering)

Oh yes, right.

I like Serena said:
Btw, I don't understand yet why it's supposed to hold that $\int_{\partial{B_R}} P(x,\xi) ds_{\xi}=1$.
Do you?
Is it somewhere in your notes?

We have that if $u \in C^2(B_R(0)) \cap C^1(\overline{B_R}(0))$ is harmonic in $B_R(0)$, then the following formula holds:

$u(x)=\frac{R^2-|x|^2}{w_n R} \int_{\partial{B_R}} \frac{u}{|x-\xi|^n} ds_{\xi}$.

For $u=1$ we get that $\int_{\partial{B_R}} P(x,\xi) ds_{\xi}=1$.

I like Serena said:
Anyway, if it holds, then, since $P(x,\xi) > 0$, we have:
$$\int_{|\xi-x_0|> \delta, \xi \in \partial{B_R}} P(x,\xi)ds \le \int_{\partial{B_R}} P(x,\xi) ds_{\xi} = 1$$
It seems we don't need it though. (Thinking)

So then we have $\int_{|\xi-x_0|> \delta, \xi \in \partial{B_R}} P(x,\xi) |\phi(\xi)-\phi(x_0)| dS \leq \sup_{\xi \in \partial{B_R}, |\xi-x_0|> \delta} |\phi(\xi)-\phi(x_0)| \int_{|\xi-x_0|>\delta, \xi \in \partial{B_R}} P(x, \xi) dS \leq 2M \int_{\partial{B_R}} P(x,\xi) dS=2M $So we have to pick $4M$ as $\epsilon$ ?
 
  • #16
evinda said:
We have that if $u \in C^2(B_R(0)) \cap C^1(\overline{B_R}(0))$ is harmonic in $B_R(0)$, then the following formula holds:

$u(x)=\frac{R^2-|x|^2}{w_n R} \int_{\partial{B_R}} \frac{u}{|x-\xi|^n} ds_{\xi}$.

For $u=1$ we get that $\int_{\partial{B_R}} P(x,\xi) ds_{\xi}=1$.

Ok! (Happy)

So then we have $\int_{|\xi-x_0|> \delta, \xi \in \partial{B_R}} P(x,\xi) |\phi(\xi)-\phi(x_0)| dS \leq \sup_{\xi \in \partial{B_R}, |\xi-x_0|> \delta} |\phi(\xi)-\phi(x_0)| \int_{|\xi-x_0|>\delta, \xi \in \partial{B_R}} P(x, \xi) dS \leq 2M \int_{\partial{B_R}} P(x,\xi) dS=2M $

So we have to pick $4M$ as $\epsilon$ ?

I don't think so. We cannot make $4M$ as small as we want to.

Shouldn't we pick $\xi$ sufficiently close to $x_0$ for the proof?
That is, $|\xi-x_0|<\delta$? (Wondering)
 
  • #17
I like Serena said:
Ok! (Happy)
I don't think so. We cannot make $4M$ as small as we want to.

Shouldn't we pick $\xi$ sufficiently close to $x_0$ for the proof?
That is, $|\xi-x_0|<\delta$? (Wondering)

We pick $|x-x_0|$ to be sufficiently small.

I think that I understood the inequality of my notes.

Since $\phi$ is bounded, there is a $M>0$ such that $|\phi| \leq M$.

So $\int_{|\xi-x_0|> \delta, \xi \in \partial{B_R}} P(x,\xi) |\phi(\xi)-\phi(x_0)| ds \leq 2M \int_{|\xi-x_0|>\delta, \xi \in \partial{B_R}} \frac{R^2-|x|^2}{w_n R |x-\xi|^n} ds < 2M \frac{R^2-|x|^2}{R} \frac{1}{\delta^n} R^{n-1}, \text{ since } |\xi-x_0|> \delta$.

Since $|x-x_0|$ is sufficiently small , $|x| \to |x_0|=|R|$ and so $\frac{R^2-|x|^2}{R} \frac{1}{\delta^n} R^{n-1} \to 0$ and so it is $\leq \frac{\epsilon}{2}$.
 
  • #18
evinda said:
We pick $|x-x_0|$ to be sufficiently small.

Oh yes. :eek:

I think that I understood the inequality of my notes.

Since $\phi$ is bounded, there is a $M>0$ such that $|\phi| \leq M$.

So $\int_{|\xi-x_0|> \delta, \xi \in \partial{B_R}} P(x,\xi) |\phi(\xi)-\phi(x_0)| ds \leq 2M \int_{|\xi-x_0|>\delta, \xi \in \partial{B_R}} \frac{R^2-|x|^2}{w_n R |x-\xi|^n} ds < 2M \frac{R^2-|x|^2}{R} \frac{1}{\delta^n} R^{n-1}, \text{ since } |\xi-x_0|> \delta$.

Since $|x-x_0|$ is sufficiently small , $|x| \to |x_0|=|R|$ and so $\frac{R^2-|x|^2}{R} \frac{1}{\delta^n} R^{n-1} \to 0$ and so it is $\leq \frac{\epsilon}{2}$.

Good! (Happy)
 

1. What does the equation $\Delta u=0$ represent?

The equation $\Delta u=0$ represents Laplace's equation, which is a partial differential equation commonly used in mathematical physics to model steady-state systems with no sources or sinks of energy.

2. What does the term "uniqueness of solution" mean?

"Uniqueness of solution" means that there is only one possible solution to the given equation that satisfies all of the specified conditions. In the case of $\Delta u=0$ in a ball with boundary condition $\phi$, this means that there is only one possible solution for the function $u(x,y,z)$ that satisfies Laplace's equation and the given boundary condition $\phi$ within the defined ball.

3. Why is uniqueness of solution important in this context?

In the context of Laplace's equation in a ball with a given boundary condition, uniqueness of solution is important because it ensures that there is only one possible solution for the system. This means that the solution is well-defined and can be used to accurately model and predict the behavior of the system.

4. How is uniqueness of solution determined for $\Delta u=0$ in a ball with boundary condition $\phi$?

Uniqueness of solution for $\Delta u=0$ in a ball with boundary condition $\phi$ is typically determined using mathematical proofs and techniques, such as the maximum principle and the method of Green's functions. These methods can show that a unique solution exists and can also help to find the specific form of the solution.

5. Are there any situations where uniqueness of solution may not hold for $\Delta u=0$ in a ball with boundary condition $\phi$?

Yes, there can be situations where uniqueness of solution may not hold. For example, if the boundary condition $\phi$ is not well-defined or if the given ball contains sources or sinks of energy, then uniqueness of solution may not hold. In these cases, additional conditions or techniques may be needed to determine a unique solution.

Similar threads

Replies
2
Views
1K
  • Differential Equations
Replies
20
Views
4K
Replies
1
Views
1K
  • Differential Equations
Replies
1
Views
1K
  • Differential Equations
Replies
5
Views
2K
  • Differential Equations
Replies
8
Views
4K
  • Differential Equations
Replies
1
Views
1K
  • Differential Equations
Replies
2
Views
1K
  • Differential Geometry
Replies
2
Views
591
Replies
5
Views
391
Back
Top